Calculating Laurent expansions using geometric series in regions defined by inequalities












1












$begingroup$


So I am a bit confused about Laurent series and their relation to geometric series. I will give an example but my doubts are more general. The function:
$$f(z) = {frac {{{rm e}^{z}}}{
left( z-1 right) ^{2}}}
$$

has a singularity at $z = 1$. I know that the function converges to a Laurent series only in the region $ left| z-1 right| > 0$ which is an "infinite circle" centered at 1 (right?). So relating to the geometric series:
$ left( z-1 right) ^{-2}$ which is obtained by differentiating $ left( 1-z right) ^{-1}$. However, this geometric series is only valid for $$ left| z right| <1$$ inside the circle with radius $ left| z right|=1$ which is not the same as $ left| z-1 right| > 0$.



My question is: how can I relate a geometric series with a correct Laurent expansion and find the correct geometric series in "the correct" regions defined by a certain inequality?










share|cite|improve this question











$endgroup$












  • $begingroup$
    Couple things: The region $|z-1| > 0$ is the region $zne 1$, it's the punctured plane. Second thing, what is an infinite circle? Also yes, you can find Taylor series for the function by centering the Taylor series around 0 as you're suggesting, and yes it's only valid in a circle with radius the distance to the nearest singularity. Also you simultaneously said the geometric series is and is not valid inside the circle $|z|=1$. The Laurent series is nicer because it's globally valid.
    $endgroup$
    – jgon
    Dec 31 '18 at 20:12










  • $begingroup$
    $frac1{1-z}=1+z+z^2+cdots$ for $|z|<1$. Then replacing $z$ with $z-1$: $frac1{1-(z-1)}=1+(z-1)+(z-1)^2+cdots$, valid for $|z-1|<1$.
    $endgroup$
    – user170231
    Dec 31 '18 at 20:23










  • $begingroup$
    @user170231 yes but that is not valid for $left| z-1 right| > 0$
    $endgroup$
    – daljit97
    Dec 31 '18 at 22:30










  • $begingroup$
    @jgon by infinite circle I mean the entire plane (excluding the singularity).
    $endgroup$
    – daljit97
    Dec 31 '18 at 22:32
















1












$begingroup$


So I am a bit confused about Laurent series and their relation to geometric series. I will give an example but my doubts are more general. The function:
$$f(z) = {frac {{{rm e}^{z}}}{
left( z-1 right) ^{2}}}
$$

has a singularity at $z = 1$. I know that the function converges to a Laurent series only in the region $ left| z-1 right| > 0$ which is an "infinite circle" centered at 1 (right?). So relating to the geometric series:
$ left( z-1 right) ^{-2}$ which is obtained by differentiating $ left( 1-z right) ^{-1}$. However, this geometric series is only valid for $$ left| z right| <1$$ inside the circle with radius $ left| z right|=1$ which is not the same as $ left| z-1 right| > 0$.



My question is: how can I relate a geometric series with a correct Laurent expansion and find the correct geometric series in "the correct" regions defined by a certain inequality?










share|cite|improve this question











$endgroup$












  • $begingroup$
    Couple things: The region $|z-1| > 0$ is the region $zne 1$, it's the punctured plane. Second thing, what is an infinite circle? Also yes, you can find Taylor series for the function by centering the Taylor series around 0 as you're suggesting, and yes it's only valid in a circle with radius the distance to the nearest singularity. Also you simultaneously said the geometric series is and is not valid inside the circle $|z|=1$. The Laurent series is nicer because it's globally valid.
    $endgroup$
    – jgon
    Dec 31 '18 at 20:12










  • $begingroup$
    $frac1{1-z}=1+z+z^2+cdots$ for $|z|<1$. Then replacing $z$ with $z-1$: $frac1{1-(z-1)}=1+(z-1)+(z-1)^2+cdots$, valid for $|z-1|<1$.
    $endgroup$
    – user170231
    Dec 31 '18 at 20:23










  • $begingroup$
    @user170231 yes but that is not valid for $left| z-1 right| > 0$
    $endgroup$
    – daljit97
    Dec 31 '18 at 22:30










  • $begingroup$
    @jgon by infinite circle I mean the entire plane (excluding the singularity).
    $endgroup$
    – daljit97
    Dec 31 '18 at 22:32














1












1








1





$begingroup$


So I am a bit confused about Laurent series and their relation to geometric series. I will give an example but my doubts are more general. The function:
$$f(z) = {frac {{{rm e}^{z}}}{
left( z-1 right) ^{2}}}
$$

has a singularity at $z = 1$. I know that the function converges to a Laurent series only in the region $ left| z-1 right| > 0$ which is an "infinite circle" centered at 1 (right?). So relating to the geometric series:
$ left( z-1 right) ^{-2}$ which is obtained by differentiating $ left( 1-z right) ^{-1}$. However, this geometric series is only valid for $$ left| z right| <1$$ inside the circle with radius $ left| z right|=1$ which is not the same as $ left| z-1 right| > 0$.



My question is: how can I relate a geometric series with a correct Laurent expansion and find the correct geometric series in "the correct" regions defined by a certain inequality?










share|cite|improve this question











$endgroup$




So I am a bit confused about Laurent series and their relation to geometric series. I will give an example but my doubts are more general. The function:
$$f(z) = {frac {{{rm e}^{z}}}{
left( z-1 right) ^{2}}}
$$

has a singularity at $z = 1$. I know that the function converges to a Laurent series only in the region $ left| z-1 right| > 0$ which is an "infinite circle" centered at 1 (right?). So relating to the geometric series:
$ left( z-1 right) ^{-2}$ which is obtained by differentiating $ left( 1-z right) ^{-1}$. However, this geometric series is only valid for $$ left| z right| <1$$ inside the circle with radius $ left| z right|=1$ which is not the same as $ left| z-1 right| > 0$.



My question is: how can I relate a geometric series with a correct Laurent expansion and find the correct geometric series in "the correct" regions defined by a certain inequality?







sequences-and-series complex-analysis laurent-series






share|cite|improve this question















share|cite|improve this question













share|cite|improve this question




share|cite|improve this question








edited Dec 31 '18 at 20:23







daljit97

















asked Dec 31 '18 at 20:02









daljit97daljit97

178111




178111












  • $begingroup$
    Couple things: The region $|z-1| > 0$ is the region $zne 1$, it's the punctured plane. Second thing, what is an infinite circle? Also yes, you can find Taylor series for the function by centering the Taylor series around 0 as you're suggesting, and yes it's only valid in a circle with radius the distance to the nearest singularity. Also you simultaneously said the geometric series is and is not valid inside the circle $|z|=1$. The Laurent series is nicer because it's globally valid.
    $endgroup$
    – jgon
    Dec 31 '18 at 20:12










  • $begingroup$
    $frac1{1-z}=1+z+z^2+cdots$ for $|z|<1$. Then replacing $z$ with $z-1$: $frac1{1-(z-1)}=1+(z-1)+(z-1)^2+cdots$, valid for $|z-1|<1$.
    $endgroup$
    – user170231
    Dec 31 '18 at 20:23










  • $begingroup$
    @user170231 yes but that is not valid for $left| z-1 right| > 0$
    $endgroup$
    – daljit97
    Dec 31 '18 at 22:30










  • $begingroup$
    @jgon by infinite circle I mean the entire plane (excluding the singularity).
    $endgroup$
    – daljit97
    Dec 31 '18 at 22:32


















  • $begingroup$
    Couple things: The region $|z-1| > 0$ is the region $zne 1$, it's the punctured plane. Second thing, what is an infinite circle? Also yes, you can find Taylor series for the function by centering the Taylor series around 0 as you're suggesting, and yes it's only valid in a circle with radius the distance to the nearest singularity. Also you simultaneously said the geometric series is and is not valid inside the circle $|z|=1$. The Laurent series is nicer because it's globally valid.
    $endgroup$
    – jgon
    Dec 31 '18 at 20:12










  • $begingroup$
    $frac1{1-z}=1+z+z^2+cdots$ for $|z|<1$. Then replacing $z$ with $z-1$: $frac1{1-(z-1)}=1+(z-1)+(z-1)^2+cdots$, valid for $|z-1|<1$.
    $endgroup$
    – user170231
    Dec 31 '18 at 20:23










  • $begingroup$
    @user170231 yes but that is not valid for $left| z-1 right| > 0$
    $endgroup$
    – daljit97
    Dec 31 '18 at 22:30










  • $begingroup$
    @jgon by infinite circle I mean the entire plane (excluding the singularity).
    $endgroup$
    – daljit97
    Dec 31 '18 at 22:32
















$begingroup$
Couple things: The region $|z-1| > 0$ is the region $zne 1$, it's the punctured plane. Second thing, what is an infinite circle? Also yes, you can find Taylor series for the function by centering the Taylor series around 0 as you're suggesting, and yes it's only valid in a circle with radius the distance to the nearest singularity. Also you simultaneously said the geometric series is and is not valid inside the circle $|z|=1$. The Laurent series is nicer because it's globally valid.
$endgroup$
– jgon
Dec 31 '18 at 20:12




$begingroup$
Couple things: The region $|z-1| > 0$ is the region $zne 1$, it's the punctured plane. Second thing, what is an infinite circle? Also yes, you can find Taylor series for the function by centering the Taylor series around 0 as you're suggesting, and yes it's only valid in a circle with radius the distance to the nearest singularity. Also you simultaneously said the geometric series is and is not valid inside the circle $|z|=1$. The Laurent series is nicer because it's globally valid.
$endgroup$
– jgon
Dec 31 '18 at 20:12












$begingroup$
$frac1{1-z}=1+z+z^2+cdots$ for $|z|<1$. Then replacing $z$ with $z-1$: $frac1{1-(z-1)}=1+(z-1)+(z-1)^2+cdots$, valid for $|z-1|<1$.
$endgroup$
– user170231
Dec 31 '18 at 20:23




$begingroup$
$frac1{1-z}=1+z+z^2+cdots$ for $|z|<1$. Then replacing $z$ with $z-1$: $frac1{1-(z-1)}=1+(z-1)+(z-1)^2+cdots$, valid for $|z-1|<1$.
$endgroup$
– user170231
Dec 31 '18 at 20:23












$begingroup$
@user170231 yes but that is not valid for $left| z-1 right| > 0$
$endgroup$
– daljit97
Dec 31 '18 at 22:30




$begingroup$
@user170231 yes but that is not valid for $left| z-1 right| > 0$
$endgroup$
– daljit97
Dec 31 '18 at 22:30












$begingroup$
@jgon by infinite circle I mean the entire plane (excluding the singularity).
$endgroup$
– daljit97
Dec 31 '18 at 22:32




$begingroup$
@jgon by infinite circle I mean the entire plane (excluding the singularity).
$endgroup$
– daljit97
Dec 31 '18 at 22:32










2 Answers
2






active

oldest

votes


















3












$begingroup$

Use the fact thatbegin{align}frac{e^z}{(z-1)^2}&=efrac{e^{z-1}}{(z-1)^2}\&=efrac{displaystylesum_{n=0}^inftyfrac{(z-1)^n}{n!}}{(z-1)^2}.end{align}






share|cite|improve this answer









$endgroup$













  • $begingroup$
    Yes that is a more "clever" way of doing this however I was more interested in using the "geometric series" approach.
    $endgroup$
    – daljit97
    Dec 31 '18 at 22:29



















0












$begingroup$

Try using: $e^z = ecdot e^{z-1}$ where $|z - 1| > 0$.






share|cite|improve this answer











$endgroup$














    Your Answer








    StackExchange.ready(function() {
    var channelOptions = {
    tags: "".split(" "),
    id: "69"
    };
    initTagRenderer("".split(" "), "".split(" "), channelOptions);

    StackExchange.using("externalEditor", function() {
    // Have to fire editor after snippets, if snippets enabled
    if (StackExchange.settings.snippets.snippetsEnabled) {
    StackExchange.using("snippets", function() {
    createEditor();
    });
    }
    else {
    createEditor();
    }
    });

    function createEditor() {
    StackExchange.prepareEditor({
    heartbeatType: 'answer',
    autoActivateHeartbeat: false,
    convertImagesToLinks: true,
    noModals: true,
    showLowRepImageUploadWarning: true,
    reputationToPostImages: 10,
    bindNavPrevention: true,
    postfix: "",
    imageUploader: {
    brandingHtml: "Powered by u003ca class="icon-imgur-white" href="https://imgur.com/"u003eu003c/au003e",
    contentPolicyHtml: "User contributions licensed under u003ca href="https://creativecommons.org/licenses/by-sa/3.0/"u003ecc by-sa 3.0 with attribution requiredu003c/au003e u003ca href="https://stackoverflow.com/legal/content-policy"u003e(content policy)u003c/au003e",
    allowUrls: true
    },
    noCode: true, onDemand: true,
    discardSelector: ".discard-answer"
    ,immediatelyShowMarkdownHelp:true
    });


    }
    });














    draft saved

    draft discarded


















    StackExchange.ready(
    function () {
    StackExchange.openid.initPostLogin('.new-post-login', 'https%3a%2f%2fmath.stackexchange.com%2fquestions%2f3058006%2fcalculating-laurent-expansions-using-geometric-series-in-regions-defined-by-ineq%23new-answer', 'question_page');
    }
    );

    Post as a guest















    Required, but never shown

























    2 Answers
    2






    active

    oldest

    votes








    2 Answers
    2






    active

    oldest

    votes









    active

    oldest

    votes






    active

    oldest

    votes









    3












    $begingroup$

    Use the fact thatbegin{align}frac{e^z}{(z-1)^2}&=efrac{e^{z-1}}{(z-1)^2}\&=efrac{displaystylesum_{n=0}^inftyfrac{(z-1)^n}{n!}}{(z-1)^2}.end{align}






    share|cite|improve this answer









    $endgroup$













    • $begingroup$
      Yes that is a more "clever" way of doing this however I was more interested in using the "geometric series" approach.
      $endgroup$
      – daljit97
      Dec 31 '18 at 22:29
















    3












    $begingroup$

    Use the fact thatbegin{align}frac{e^z}{(z-1)^2}&=efrac{e^{z-1}}{(z-1)^2}\&=efrac{displaystylesum_{n=0}^inftyfrac{(z-1)^n}{n!}}{(z-1)^2}.end{align}






    share|cite|improve this answer









    $endgroup$













    • $begingroup$
      Yes that is a more "clever" way of doing this however I was more interested in using the "geometric series" approach.
      $endgroup$
      – daljit97
      Dec 31 '18 at 22:29














    3












    3








    3





    $begingroup$

    Use the fact thatbegin{align}frac{e^z}{(z-1)^2}&=efrac{e^{z-1}}{(z-1)^2}\&=efrac{displaystylesum_{n=0}^inftyfrac{(z-1)^n}{n!}}{(z-1)^2}.end{align}






    share|cite|improve this answer









    $endgroup$



    Use the fact thatbegin{align}frac{e^z}{(z-1)^2}&=efrac{e^{z-1}}{(z-1)^2}\&=efrac{displaystylesum_{n=0}^inftyfrac{(z-1)^n}{n!}}{(z-1)^2}.end{align}







    share|cite|improve this answer












    share|cite|improve this answer



    share|cite|improve this answer










    answered Dec 31 '18 at 20:14









    José Carlos SantosJosé Carlos Santos

    176k24136245




    176k24136245












    • $begingroup$
      Yes that is a more "clever" way of doing this however I was more interested in using the "geometric series" approach.
      $endgroup$
      – daljit97
      Dec 31 '18 at 22:29


















    • $begingroup$
      Yes that is a more "clever" way of doing this however I was more interested in using the "geometric series" approach.
      $endgroup$
      – daljit97
      Dec 31 '18 at 22:29
















    $begingroup$
    Yes that is a more "clever" way of doing this however I was more interested in using the "geometric series" approach.
    $endgroup$
    – daljit97
    Dec 31 '18 at 22:29




    $begingroup$
    Yes that is a more "clever" way of doing this however I was more interested in using the "geometric series" approach.
    $endgroup$
    – daljit97
    Dec 31 '18 at 22:29











    0












    $begingroup$

    Try using: $e^z = ecdot e^{z-1}$ where $|z - 1| > 0$.






    share|cite|improve this answer











    $endgroup$


















      0












      $begingroup$

      Try using: $e^z = ecdot e^{z-1}$ where $|z - 1| > 0$.






      share|cite|improve this answer











      $endgroup$
















        0












        0








        0





        $begingroup$

        Try using: $e^z = ecdot e^{z-1}$ where $|z - 1| > 0$.






        share|cite|improve this answer











        $endgroup$



        Try using: $e^z = ecdot e^{z-1}$ where $|z - 1| > 0$.







        share|cite|improve this answer














        share|cite|improve this answer



        share|cite|improve this answer








        edited Dec 31 '18 at 21:13









        Davide Giraudo

        128k17156268




        128k17156268










        answered Dec 31 '18 at 20:19









        Swapnil Swapnil

        515




        515






























            draft saved

            draft discarded




















































            Thanks for contributing an answer to Mathematics Stack Exchange!


            • Please be sure to answer the question. Provide details and share your research!

            But avoid



            • Asking for help, clarification, or responding to other answers.

            • Making statements based on opinion; back them up with references or personal experience.


            Use MathJax to format equations. MathJax reference.


            To learn more, see our tips on writing great answers.




            draft saved


            draft discarded














            StackExchange.ready(
            function () {
            StackExchange.openid.initPostLogin('.new-post-login', 'https%3a%2f%2fmath.stackexchange.com%2fquestions%2f3058006%2fcalculating-laurent-expansions-using-geometric-series-in-regions-defined-by-ineq%23new-answer', 'question_page');
            }
            );

            Post as a guest















            Required, but never shown





















































            Required, but never shown














            Required, but never shown












            Required, but never shown







            Required, but never shown

































            Required, but never shown














            Required, but never shown












            Required, but never shown







            Required, but never shown







            Popular posts from this blog

            Biblatex bibliography style without URLs when DOI exists (in Overleaf with Zotero bibliography)

            ComboBox Display Member on multiple fields

            Is it possible to collect Nectar points via Trainline?